LSAT and Law School Admissions Forum

Get expert LSAT preparation and law school admissions advice from PowerScore Test Preparation.

 gen2871
  • Posts: 47
  • Joined: Jul 01, 2018
|
#47925
Hi Nikki,

I dont quite understand how you come up with this diagram Weak Economy :arrow: (price constant + unemployment goes up).

It says if the economy is weak, then the price remain constant although unemployment rises. So my diagram goes "weak economy :arrow: price :longline:. But I dont really know how to put Although the unemployment went up. Seems to me that although is a sufficient indicator, and you placed it in the necessary condition slot.

Please help me understand this. Thank you!
 LSAT2018
  • Posts: 242
  • Joined: Jan 10, 2018
|
#48841
How does this "even if" clause affect the conditional rule we outlined above? The answer, oddly enough, is that it doesn't really affect it at all. This clause is simply stating that the conditional rule above holds true regardless of whether the descriptions are identified as opinions. In other words, the fact that the auction house identifies these descriptions as opinions does not negate the rule in any way.
I read in one of your blog posts about 'even if' in conditional reasoning. I read through your explanations and understand why you have diagrammed it as a compound conditional statement. But I just wanted to clarify why the 'although' in the conditional statement is different from 'even if' in other cases. When I first read through, I disregarded the 'although unemployment rises' thinking it is an 'even if' clause.

What are your thoughts on this?
 Adam Tyson
PowerScore Staff
  • PowerScore Staff
  • Posts: 5153
  • Joined: Apr 14, 2011
|
#49543
"Although" is NOT a conditional indicator, guys. It is a premise indicator, and just means "here comes some evidence that may appear to conflict with other evidence." In this argument, although means "and also" - prices remain constant and also investment decreases. Those two necessary conditions in the presence of a weak economy may seem to be at odds with each other at some level, which is why the author used "although," but they are apparently consistent.

"Although" also does not mean the same thing as "even if," although it could be used that way in certain contexts. "Even if" is used to challenge a necessary condition, to show that it is not truly necessary. "You say that a 180 on the LSAT is necessary in order to get into Harvard Law with a full scholarship, but I know you can get into Harvard Law with a full scholarship even if you don't get a 180, because my cousin Mandy did and she scored a 174." I could use "although" in that argument in the same way - "You say that a 180 on the LSAT is necessary in order to get into Harvard Law with a full scholarship, although my cousin Mandy did and she scored a 174, so you're wrong." But I could also use "although" in a completely different way: A 180 on the LSAT is necessary in order to get into Harvard Law with a full scholarship, although you can get in with a partial scholarship with anything over a 172." In the latter case, "although" does nothing to challenge the original conditional claim, but just adds some more evidence that might mitigate or otherwise differ from the first bit of evidence.

In this argument, "although" is most definitely NOT a conditional indicator or challenge like "even if" because it isn't saying that something isn't necessary. It's just being used here as an "and also" indicator.

In short, "even if" means something is NOT necessary, while "although" means "here's something different."

Let us know if that doesn't clear it up - or even if it does!
 gen2871
  • Posts: 47
  • Joined: Jul 01, 2018
|
#50004
Hi Adam, thank you for the explanation. Its been over a month since I initially posted this question, and it looks extremely silly on why I posed it back then. I guess it is because I have made quite a lot progress so that I could understand a lot of the stuff which I had no idea back then.

Anyways, don't mean to get sentimental here, but thank you and your lovely coworkers for the unconditional love and support you guys have been dedicating! None of us would be able to make the progress we make now without your help!
 phoenixflame
  • Posts: 2
  • Joined: Aug 27, 2018
|
#50231
I am not sure how to diagram the stimulus. The last premise through me off.

EW (economy is weak) :arrow: PC (prices remain constant) :arrow: UR (although unemployment rises) :arrow: ID

This is the contrapositive:

ID :arrow: UR :arrow: PC :arrow: EW

I did not know how to factor in the both "although unemployment rises" and "fortunately investment is not decreasing.

I chose E because I thought it was the best answer. I do not understand how I got the question wrong. I want to ask if you are able to explain the answer and tell me if I diagrammed the stimulus correctly.
 James Finch
PowerScore Staff
  • PowerScore Staff
  • Posts: 943
  • Joined: Sep 06, 2017
|
#50446
Hi Phoenix,

I would diagram the "although unemployment rises" as an additional necessary condition of a weak economy, so that the diagram would look like:

Weak Economy (WE) :arrow: Prices Constant (PC) + Unemployment Rises (UR)

Then we have UR :arrow: Investment Decreases (ID). So the whole diagram would look something like:

WE :arrow: PC + UR :arrow: ID

Plus the contrapositive:

ID :arrow: UR, UR or PC :arrow: WE

So WE :arrow: ID and ID:arrow: WE

Then we are tasked by the question stem to find which answer must be false (ie cannot be true), which in this case means which two conditions cannot exist together at the same time. (A) works because whenever the economy is weak, investment decreases, so it can't be an either/or scenario. (E) is possible, because you could have a situation where unemployment is rising but prices aren't constant, so the economy wouldn't be weak.

Hope this clears things up!
 lenihil
  • Posts: 35
  • Joined: Apr 27, 2020
|
#78005
Let me try this one.

Many students are confused by: 1) How to diagram the answers correctly; 2) What's going on with answer (A) and (D).

1) Diagrams

(A) -EW -> ID or -ID -> EW
(B) UR -> PRC
(C) EW -> ID
(D) -EW -> PRC or -PRC -> EW
(E) -UR -> -EW or EW -> UR

2) Choose (A) or (D)

What the stimulus told us:
1. EW -> PRC
EW -> UR -> ID
and don't forget we're also told:
2. -ID
from which we can infer:
-ID -> -UR ->-EW
This is the key to deciding whether answer choice (A) or (D) is the correct must be false answer.



Les't talk about answer (A) and (D):

(A) -EW -> ID or -ID -> EW

The first part -EW -> ID could be true at first. It is consistent with EW -> UR -> ID. We only know that when the economy is weak, investment decreases. But we don't know what if economy is not weak. So, they are consistent with each other. There is no violation. Except that we are told -ID is true now. The first part tells us ID is true. They are in conflict. So the first part cannot be true.

The second part -ID -> EW cannot be true. Because we are told in the stimulus that EW -> UR -> ID. Taking contrapositive of this, we know -ID -> -UR -> -EW. That is -ID -> -EW, which contradict the second part -ID -> EW.

Now, both parts of answer choice (A) cannot be true. It's the correct answer.

(D) -EW -> PRC or -PRC -> EW

The first part -EW -> PRC could be true. It is consistent with EW -> PRC. We only know that when the economy is weak, prices remain constant. But we don't know what if economy is not weak. So, they are consistent with each other.

The second part -PRC -> EW cannot be true. Because we are told in the stimulus that EW -> PRC. Taking contrapositive of this, we know -PRC -> -EW, which contradict the second part -PRC -> EW.

Which one should we choose? We are given -ID, which carries -ID -> -UR -> -EW with it. So, we can infer three facts given: -ID, -UR and -EW. Now it's clear, we should accept -EW as the sufficient condition, which means we should choose the first part. Because we are never told that -PRC is true, we don't know if the second part should apply. So, answer (D) could be true.

Or, you can think it in this way: Although the second part cannot be true, but the first could still work. It is not totally false. Under some circumstance (when -EW is true), it could be true. So, we cannot say answer choice (D) is definitely false.

Answer choice (A) is the right answer.

Hope this helps. Good luck on LSAT.:ras: :ras:
User avatar
 PresidentLSAT
  • Posts: 87
  • Joined: Apr 19, 2021
|
#90461
Gosh this question was difficult lol. Someone please help with my questions.

I didn't know how to diagram the 'although' alongside prices remaining constant. I figured both went hand in hand but couldn't notate it. Does although always mean '+"?

Also, if If the first sentence diagrams, Weak----> Prices constant, then why does the second
sentence also diagram
Unemployment Rises----> Investment Decreases ? Respectively, they read, A ----> B, B---->A. How come the responses diagram them in the same way?

Ok, back to my stimulus

Premise 1: WE----> PC + UR
-(PC +UR) ----> -WE

Premise 2: UR ----> ID
- ID ----> -UR (Does it mean that if Unemployment isn't rising as a result of no reduction in investment, then prices are constant?) -ID----> -(UR + PC)?


Conclusion: -ID

Combining Premise 1 & 2: -ID ----> -UR ----> -WE
Contrapositive : WE ----> UR ----> ID? Can we deduce this?

orrrrrrrr

-ID ----> -(UR +PC) ----> -WE
WE ----> UR + PC ------> WE

A. Per the information we have, -ID ----> -W (which means the economy can't be weak). The 2nd part is impossible because it's direct contraction to the stimulus.

B. Me thinks because they happen simultaneously, it's fair to deduce A->B

C. Like my question earlier, I'm not sure if this is diagrammed WE----> ID or ID ---->WE
I can infer answer C from the contrapositive (1st choice) of my conclusion.

D. How is D possible? If we have a weak economy, prices are constant. Does either or goes against the information we have? Can we have a weak economy and not have constant prices? How is that possible when weak economy halts prices by default?

E. UI or -W? How do we deduce this? If investment isn't increasing, unemployment isn't rising, per the combination of the stimulus
User avatar
 evelineliu
PowerScore Staff
  • PowerScore Staff
  • Posts: 91
  • Joined: Sep 06, 2021
|
#90488
Hi PresidentLSAT,

Sentence 1 says that a weak economy is sufficient to bring about 2 effects: constant prices & rising unemployment, which can be diagrammed as:
  • Economy is weak --> Prices are constant AND rising unemployment


Sentence 2 translates to:
  • If unemployment rises --> investment decreases.
The contrapositive is:
  • If investment NOT decreasing --> unemployment NOT rising
Sentence 3 says investment is not decreasing. From Sentence 2's contrapositive: we can therefore determine that unemployment is not rising. This has an impact on Sentence 1, which has the following contrapositive:
  • If either prices are NOT constant OR unemployment is NOT rising --> the economy is NOT weak.
The final deduction is the economy is not weak.

(A) is impossible.

Hope that helps,
Eveline

Get the most out of your LSAT Prep Plus subscription.

Analyze and track your performance with our Testing and Analytics Package.